LSAT and Law School Admissions Forum

Get expert LSAT preparation and law school admissions advice from PowerScore Test Preparation.

User avatar
 Dave Killoran
PowerScore Staff
  • PowerScore Staff
  • Posts: 5852
  • Joined: Mar 25, 2011
|
#27293
Complete Question Explanation
(The complete setup for this game can be found here: lsat/viewtopic.php?t=11501)

The correct answer choice is (C)

If you understood the principle at work in question #5, this question should feel relatively comfortable.

If lockers 1 and 2 are single lockers assigned to boys, they must be assigned to M and P, not necessarily in that order. The shared lockers must then be assigned to lockers 3, 4, or 5. Because N and T are assigned shared lockers that cannot be adjacent lockers, they must be assigned to lockers 3 and 5, not necessarily in that order. R must then be assigned to locker 4. And, because F is already assigned to locker 3, J must then be assigned to locker 5, leading to the following scenario:
June 00_M12_game#1_L9_explanations_game#2_#6_diagram_1.png
Accordingly, answer choice (C) is correct.
You do not have the required permissions to view the files attached to this post.

Get the most out of your LSAT Prep Plus subscription.

Analyze and track your performance with our Testing and Analytics Package.